Difference between revisions of "2009 AMC 8 Problems/Problem 13"

(Created page with "==Problem== A three-digit integer contains one of each of the digits <math> 1</math>, <math> 3</math>, and <math> 5</math>. What is the probability that the integer is divisible...")
 
Line 9: Line 9:
 
\textbf{(D)}\  \frac{2}{3}  \qquad
 
\textbf{(D)}\  \frac{2}{3}  \qquad
 
\textbf{(E)}\  \frac{5}{6}</math>
 
\textbf{(E)}\  \frac{5}{6}</math>
 +
 +
==See Also==
 +
{{AMC8 box|year=2009|num-b=11|num-a=12}}

Revision as of 17:51, 5 November 2012

Problem

A three-digit integer contains one of each of the digits $1$, $3$, and $5$. What is the probability that the integer is divisible by $5$?


$\textbf{(A)}\  \frac{1}{6}  \qquad \textbf{(B)}\   \frac{1}{3}  \qquad \textbf{(C)}\   \frac{1}{2}  \qquad \textbf{(D)}\  \frac{2}{3}   \qquad \textbf{(E)}\   \frac{5}{6}$

See Also

2009 AMC 8 (ProblemsAnswer KeyResources)
Preceded by
Problem 11
Followed by
Problem 12
1 2 3 4 5 6 7 8 9 10 11 12 13 14 15 16 17 18 19 20 21 22 23 24 25
All AJHSME/AMC 8 Problems and Solutions